You are on page 1of 59

quants concepts

Compiled by Amul Badjatya From pagal guy threads-


=====================================================================32^32=(1024)^
16=(24)^16= (576)^8=(24)^8=(24)^4=24^2=576 .Hence second last digit=7.

=========================

An extension of the same problem:


1111.......n times is a composite number for sure only when n is composite number.

==================================
oday's funda...it's a combination of few of my old posts...

factorial based questions asking no. of zeroes and max power of sum integer.

Find the no. of zeroes at the right end of 300!

for every zero, we require 10..n every 10 is made up of 5x2.


in the expression 1x2x3...300, multiples of 2 wud obviously be more than the
multiples of 5...so v need to find the maximum power of 5 in 300!

300/5 = 60 (because every fifth no. is a multiple of 5)

300/25 = 12(because every mutiple of 25 has two 5s in it) or, 60/5=12

300/125 = 3 (because multiples of 125 have three 5s in it) or,


12/5 = 2

now 2 cannot be further divided by 5 so add all the quotients...60 + 12 + 2 = 74.

we might also get the same type of questions in a different form,

500! is divisible by 1000^n...what is the max. integral value of n?

now every 1000 is made up of 3 5s and 3 2s....2s are redundant...we need to count
no. of 5s....so find total no. of 5s and divide by 3

500/5 = 100
100/5 = 20
20/5 = 4

100 + 20 + 4 =124

124/3 = 41.33

max integral value is 41.

500! is divisible by 99^n...what is the max. integral value of n?

now every 99 is made of two 3s and one 11. obviously 11 will be the deciding
factor. so count no. of 11s for the answer
500/11 = 45
45/11 = 4

ans will be 49.

so in such questions, just check which prime no. will be the deciding factor and
count the no. of times it occurs. but please understand that highest prime no. is
not necessarily always the deciding factor. see this example:

100! is divisible by 160^n...what is the max. integral value of n?

now 160 = 2^5 * 5^1. now although 5 is the biggest prime no. that 160 is made of,
the deciding factor wud be 2. because five 2s occur less often than one 5 does. so
we'll count the no. of 2s and divide by 5.

100/2 = 50
50/2 = 25
25/2 =12
12/2 = 6
6 /2 = 3
3/2 = 1

add 'em all...97.

97/5 = 19.

so the answer wud be 19

had v taken 5 as the deciding factor, the answer wud have been 100/5 + 100/25 = 24
which is more than 19...hence a wrong answer...

when in dilemma as to which prime no. wud be the deciding factor (e.g. a divisor
like 144...its not possible to decide whether 3 or 2 will give the right
answer) ....take out answer using both the prime nos...the one thats less is the
right answer.

50! is divisible by 144^n...what is the max. integral value of n?

144 = 2^4 * 3^2...difficult to decide whether 3 or 2 will be the deciding


factor...

count 2s

50/2=25
25/2=12
12/2=6
6/2=3
3/2=1

sum=47

answer = 47/4 = 11.

count 3s

50/3=17
17/3=5
5/3=1

sum = 23

23/2 = 11

a tie...else the smaller value wud have been the answer.

300! is divisible by (24!)^n. what is the max. possible integral value of n?

such questions are tricky...when u expand 24!...u get 1x2x3...24.

in this range the highest prime no. is 23...so maximum power of 23 in 300! will
decide the max value of x...

when v expand 300!...v get a 23 in 23, 46,69,92....

total no of multiples of 23 in 300! will be 300/23 = 13,

forget the fractional part. so the maximum possible answer is 13. hope am
clear...else, feel free to revert.

256! is expanded and expressed in base 576 . how many zeroes will this expression
have on its right end?

such questions are same as finding maximum power of 576 in 256!

576 = 2^6 x 3^2


to get six 2s i have to travel eight places...1x2x3x4x5x6x7x8 has seven 2s. but to
two 3s i have to travel only six places...1x2x3...6 has two 3s...hence 2 will be
the constrain.

total 2s in 256! = 255

hence, no. of zeroes = 256/6 = 42.

just to check...3s = 126, 126/2 = 63>42

ans-42

Questions based on this concept

400! is divisible by x^n. what is the max. possible integral value of n if the
value of x is:

Q1. 300
Q2. 99
Q3. 500
Q4. 320
Q5. 770
Q6. 5200
Q7. 270
Q8. 686
Q9. 338
Q10. 13000

Answers... 49, 39, 33, 66, 39, 32, 65, 22, 16, 32 (the answer is not 33, this one
is actually tricky! )

200! is divisible by (x!)^n...whats the max. possible value of n when x =

Q11. 25
12. 35
13. 50
14. 100
15. 70
16. 300
17.15

answers... 8, 6, 4, 2, 3, 0, 16

300! is expanded and expressed in base x. find the no. of zeroes at the right end
of this expression when x=

18. 25
19. 15
20. 35
21. 39
22. 98

answers...37, 74, 48, 23, 24

do lemme know if there's any problem at all.

cheers!

maxximus
============================
Today's concept: Finding out smallest no. which leaves specific remainders with
specific divisors.

Type # 1.

find smallest no. other than k, that leaves remainder k when divided by w,x,y...

to solve such questions, take lcm of w,x,y...and add k to it.

e.g. find Smallest no. other than 4, that leaves remainder 4 when divided by 6,7,8
or 9...

take lcm of 6,7,8,9 and add 4

i.e. 504 + 4 = 508

Type # 2

find smallest no. that leaves remainder 3,5,7 when divided by 4,6,8 respectively.

unlike last case, this time the remainder is not constant. but if u see carefully,
difference b/w divisor n remainder is constant. i.e. 4-3=6-5=8-7=1
in such questions, take lcm of divisors n subtract the common difference from it

here, the answer wud be lcm of 4,6,8 i.e 24 - 1 = 23

Type # 3

Smallest no. that leaves remainder 3,4,5 whn divided by 5,6,7 respectively and
leaves remainder 1 with 11,

we have just seen a way to tackle the first 3 conditions...the no. wud be lcm of
5,6,7 - 2 = 208

now we have one more condition...remainder 1 with 11.

concept => to a no. if v add lcm of divisors...the corresponding remainders dont


change.

i.e to 208, if v keep adding 210 ... the first 3 conditions will continue being
fulfilled.

so, let 208 + 210k be the no. that will satisfy the 4th condition...viz (208 +
210k)% 11 = 1

208%11 = 10

210k%11 = k

therefore, 10 + k shud leave remainder 1 when divided by 11.

hence, k = 2. and the no. is 208 + 210 x 2 = 628

e.g. find the smallest no. that leaves remainder 2 when divided by 3,4 or 5 and is
divisible by 7

for first 3 conditions....no. is 120 + 2 = 122

hence, 122 + 120k is the required no. which reduces to 3 + 2k when divided by
7...now 3+2k shud be a multiple of 7...easily, k=2 and the required no. is 122 +
120 x 2 = 362

Type # 4

What if there is no relation between divisors n remainders?

e.g. find the smallest no. that leaves remainders 1 with 5, 4 with 7, 6 with 11
and 7 with 13.

we can c...there's no relation among these divisor-remainder sets...neither is the


remainder constant...nor is the difference b/w divisor n remainder a constant.

in such cases...take 1 case n target another case...


e.g. i take the case 7 with 13...and target 6 with 11.

which is the smallest no. that leaves 7 with 13? 7 itself...right? so all nos of
the form 7 + 13k will give 7 rem with 13.
now am targeting 6 with 11...so i divide 7 + 13k by 11...i get remainder 7 +
2k...now 7 + 2k = 6,17,28,39,50...so that the remainder with 11 is 6.

a no. that gives integral value of k is 17 i.e. 7 + 2k = 17. hence, k =5 and the
no. that satisfies these two conditions is 7 + 13 x 5 = 72

now that 2 conditions are fulfilled, lets target a third condition...say 4 with 7.

to 72, if v add lcm of 11, 13 i.e 143, 2 conditions awready satisfied wud continue
being satisfied...

hence the no. is of the form 72 + 143 k.

72 + 143k % 7 = 2 + 3k

now 2 + 3k shud be = 4,11,18,25,32... to satisfy the condition of 4 rem with 7..

a no. that gives integral soln is 11..i.e. 2 + 3k = 11, k = 3.

hence, the no. that satisfies all 3 conditions is 72 + 143 x 3 = 501.

now if v see carefully...4th condition...remainder 1 with 501 has already been


satisfied...so the no. v have been looking for is 501.

For ease of calculation, start from biggest divisor n gradually move to smaller
ones...u'll always see that last 1-2 conditions will be satisfied automatically.

there are theorems for solving above questions...viz chinese theorem etc...but i
solve such questions by the way i've suggested...i find this approach very
practical as the flow of nos. is very much visible...n i believe i can tackle any
twist in the question devised by cat makers thru this method...there are lotsa
other questions based on this concept which i'll soon post but the basic concept
remains the same...

few points to be noted

*you can always re-check ur answer

**at times, u can use options to solve such questions.

***dont let concepts go away believing such questions can be easily dealt with
thru options...the question may not always be find the smallest no. which...... at
times it may be ..find the sum of integers of smallest no. which leaves
remainders...blah blah...

****there may be a case when they put an option which satisfies all the conditions
but is not the smallest poss value...n put another option...our favorite...none of
these!!! lets not undermine genius of cat makers!!

questions for practice...

find the smallest no. that leaves remainder (s)

Q1. 2 when divided by 3,5,6 or 9 (other than 2)


Q2. 2,5,7 when divided by 7,10 and 12 respectively
Q3. 1,2,3,4 with 3,4,5,7 respectively.
Q4. 6 with 7,8,9,10 and 3 with 11.
Q5. 3 with 6, 0 with 11, 3 with 5, 7 with 8
Q6. 2 with 5, 7 with 8, 3 with 4, 5 with 7&11
Q7. 1 with 11, 4 with 5, 9 with 10, 7 with 9.

hope the post helps...puys...come up with ur own...sweet methods...which are


confined in ur sharp brains...i'll also appreciate if u can come up with
feedbacks/suggestions...like today i thought its better to give answers a day
late...

regards
maxximus
==============================

Hi Puys!
Here is something i used and doubt if many are aware of... This is a method to
divide quickly (very Quickly) and get results with amazing accuracy...
But let me forewarn you, it needs good amount of practice before you can even
think of trying it in exam hall...
And yes, this is not my creation. Credits to Rahul and company who shared this
with me 2 yrs ago

so here we Go!!!

The approach would be to get the denominator to either a 100 or a 1000 because
that is what percentages is all bout.
Simply focus on the fact that how do the given denominators reach 100/1000. I have
left some blank because they are very obvious.

The following are the stations between 100 and 1000

100
111 - Reduce 10%
125 - Multiply by 8
133 - Reduce 1/4
150 - Reduce 1/3
166 - Multiply by 6
182 - Add 10%
200
222 - Reduce 10%
250
273 - Add 10%
300
333 - Reduce 10%
364
400
455 - Add 10%
500
555 - Reduce 10%
600
666 - Add Half
700
750 - Add 1/3
800
833 - Add 20%
875 - Add 1/7
900
910 - Add 9%
950 - Add 5%
1000

So Task (1) you have to mug up the values of these stations. It is very important
that these values are memorized because this will help you in
knowing which number to reach from any given number. e.g. if the denominator is
887 you know you have to reach for 900 or 875 and so on.

Task (2) Practice!

Below is an approach to tackle (three digit /three digit) with consummate ease. We
shall attempt to understand it with examples.

Example 1:
What is 145/182 ------------------79.5

Steps
1. add 10% to numerator and denominator...
2. it becomes 159/200 ...............which is 79.5
( the answer from the calculator is 79.6)

Example 2:
123/178....???

Step1:- which station is closest to it?.............................200?...or some


say 150. Either is good. (identification took 2 seconds)

Step 2:- what do I have to do to go from 178 to 200/150..................add


22/subtract 28... (another 2 seconds.)

Step 3:- so if I add 22................i am actually adding slightly less than 13%
of 178 to itself.

This part is tricky.. here is how I got it:

This is how u need to think---


Our number is 178. Thus, 10% of it is 17.8
22 definitely more than 10% .
if 10% = 17.8 then 5% = 8.xx

And 2.5% = 4.xxx (don't even bother to calculate xxx)


which means its around 12.5........or 13 or 12..........

Add same to numerator...........10% of 123 = 12.3


1% = 1.23 so 2% = 2.46

So 12% = 15?
So it becomes 138/200 = 69%?
(calculator answer is 69.1).

All you require to calculate is what is 10% 1%...and approx stuff...any damn
calculation works in less than 8 seconds.:satisfie:

Some thing like (2456*4567 - 2134*3214)/2134*3214 will taken 10 seconds maximum...


it works coz I had that time 1.5 years ago..and I aint kidding..
Lets take one more.
Example 3: 532/745?
Tough???

This's how it can be approached:


# Nearest station.........750.................so add 5..............about 1% or
less..add 1% damnit
# Numerator now is 537.............(added 1%)
# Fraction is 537/750 add 1/3 each...........it is 71 something.

Remember here don't even attempt to do...537/3. Because denominator is a 1000 and
not a 100. So one digit is redundant. So all I do is........53/3
approx..18...plus..53 = 71%..(will do if answer are spaced...)

answers not spaced? Then 537 +537/3 = 537+179 = 716 which makes it 71.6
(calculator is 71.5)

Dont worry if the last few statements were difficult to digest. Try solving a few
questions and you'll get the crux of it.

Remember in CAT we don't find answers... we choose them!


Happy Computing

Libocta
----------------------------------------------------------------------
Once an IIMCian, an IIMCian for lifetime!
=================================================================

@ cat 2007 ...cool question bro...wud bring in a new concept...thats wat we're
look'n for...to let our brains work in unexplored areas...this question helps us c
another aspect while approaching a question...

here's the solution...or lets say approach...

4/3x22/7xR^3 + 4/3x22/7x r^3 is the sum of volumes...the terms in denominator shud


be cancelled with R^3 as well as r^3. but terms in numerator cannot be cancelled
since radii are integers...

hence each term is divisible by 22x4 i.e. 88 and so is the sum...

as all the options are divisible by 8 (since last 3 terms of each value is
divisible by
we just need to check which option is divisible by 11...thats option 2. so that
must be the correct option....

am i right cat 2007? do keep pouring in....

@ libocta...great to have u around...ur expertise wud be very helpful!!

with warm regards


maxximus
============================================================

Fisrt of all lets find out whats the cube of all single digit numbers
1 cube = 1
2 cube = 8
3 cube = 27
4 cube = 64
5 cube = 125
6 cube = 216
7 cube = 343
8 cube = 512
9 cube = 729
now, from the question we see that last digit of the number 328509 is 9
so the number whose cube is 328509 should end with 9 (since only cube of 9 ends
with 9)
also, 50 cube = 125000
60 cube = 216000
70 cube = 343000
our number lies between 60 cube and 70 cube, so the 1st digit has to be 6, and we
already have the last digit as 9
so the number is 69.

++++++++++++++++++++++++++========================================

a^n - b^n is always divisible by a-b

a^n - b^n is divisible by a+b when n is even.

a^n + b ^n is divisible by a+b when n is odd.

a^n + b^n + c^n +... is divisible by a+b+c+.... when n is odd.

when last n digits of a no. are divided by 2^n, the remainder is same as the
remainder when the entire no. is divided by 2^n.

Answers to yesterday's questions.

1. smallest no. that must be added to 1000 so that the resultant no. leaves
remainders 2,3,4,5 with 5,6,7,11 respectivelt----667

2. smallest no. that leaves remainders 1,2,5,6, when divided successively by


2,3,4,23.----such a no. doesnt exist as 5 rem with aint possible.

3. smallest no. that leaves remainders 4 everytime when successively divided by


7,5,10,13 respectively.----872

4. a no. leaves remainders 2,5,3,7 when successively divided by 3,7,6,9. what is


the remainder when this no. is divided by 126?----80

5.find the largest no. that leaves same remainder when it divides
2345,7645,9845,6595 and 10095.----50
6.a no. when divided 88 leaves remainder 3. what is the remainder when its divided
by 11? ----3

7.a no. when divided by 391 leaves rem. of 49. find the remainder when its divided
by 39...options 29,10,none of these,cannot be determined.---cannot be detrmnd

kudos to rockeezee who got most of them right!

questions for today...

1. 32^23 + 17^23 is definetly divisible by....

a. 49 b. 15 c. 49 & 15 d. none of these.

2. 32^23 - 17^23 is definetly divisible by....

a. 49 b. 15 c. 49 & 15 d. none of these.

3. 32^232 + 17^232 is definetly divisible by....

a. 49 b. 15 c. 49 & 15 d. none of these.

4. 32^232 - 17^232 is definetly divisible by....

a. 49 b. 15 c. 49 & 15 d. none of these.

5. 32^2321 + 17^2321 + 13^2321 + 18^2321 is definetly divisible by....

a. 40 b. 20 c. 80 d. all of these. e. none of these.

what is the remainder when 42527152653425416242624272427215287 is divided by :

6. 16

7. 32

8. 64

regards
maxximus

=================================
Today's concept...to solve tricky questions based on no. of factors of a number.

Despite the interest shown in the concept yesterday, not many cud solve the
question efficiently...so, best concept for the day can be to discuss yesterday's
problems...have a quick, practical solution to them...and practice a lot of
similar, different questions on same concept. here we go...

Find the smallest no. that has exactly.....

1. 16 factors

2. 12 factors

3. 60 factors

questions are lil tricky...but if u get the concept...they become child's


play...see how...

16 factors...that mean product of (powers +1) of all the prime nos = 16.

now 16 can be achieved in following ways...

16
8x2
4x4
2x4x2
2x2x2x2.

by sheer common sense, v can say the highest power shud go to the smallest prime
no. i.e. 2....and as v proceed, smaller powers shud be given to higher prime nos.

Concept:

powers shud reduce and the corresponding prime nos. shud increase.

so...

16...the no. is 2^15

8x2...the no. is 2^7 x 3

4x4...the no. is 2^3 x 3^3.

2x4x2...the no. is 2^3 x 3 x 5

2x2x2x2.....the no. is 2x3x5x7.

first three nos can be easily discarded as they are too big...just calculate last
two nos, they are 120 and 210...120 is smaller and hence the answer.

important: please avoid craming...in few cases last way might give best answer..in
other cases, 2nd last one...its always advicable to form patterns n check the
closer ones.

2. 12 factors
12 = 12 or 4x3 or 2x2x3. easily answer wud be 2^3 x 3^2 or 2^2 x 3 x 5

the corresponding values are 72 n 60. hence, the answer is 60.

similarly, Q3 also.

4. how many factors of 27000 are perfect cubes?

i realy wonder y nobody cud get this right...u just need to form combinations and
check which combinations give u cubes...they are 27000,27,1000, etc. am not
discussing this question...i hope when i give a similar question today...i get few
correct answers.

This question is still open for answers/discussion (so are others...but if u can
answer this...with an xplanation, it'll be gr8 )

how many A.Ps are possible such that first term is 1235 and the last term is 3535
and there are atleast...

5. 3 terms

6. 4 terms

7. 5 terms

8. 6 terms

concept:

to have first n last term as the given terms...the difference b/w the two terms
shud be divisible by the common difference. so u basically have to find how many
such common differences exist...for every common difference...u'll get a new AP.

for practice, lets take a small interval [1,15]

the difference is 15-1=14. now 14 is divisible by 1,2,7,14...four different


integers... so v can have four different APs...if v take a common difference other
than these four values...the last term wont lie in that AP.

e.g if v take the common diff = 4, the AP wud be 1,5,9,13,17...see 15 didnt lie...

so, 4 APs are possible.

now if a conditions is attached...there shud be atleast three terms...it means


that AP with 2 terms shud be neglected...
1,15 is an AP with jus 2 terms...so it shud be beglected...remember, an AP with 2
terms always lies in any interval.

so the answer wud be 4-1 = 3

if the condition is atleast four terms...then the AP with 2 terms as well as the
AP with 3 terms shud be neglected.
we know that an AP with 2 terms is bound to exist...lets c if an AP with 3 terms
also exists.

an AP with 3 terms will look like... 1, x, 15

see...there are 2 intervals... x-1 and 15-x. hence for a 3 term AP to exist, the
difference shud be divisible by 2.

since 14 is divisible by 2, we further reduce the answer by 1...so APs with


atleast 4 terms are 3-1 =2

now, if the conditions is...atleast 5 terms...v need to check if AP with 4 terms


exists...
such an AP wud luk like...

1, x, y, 15

see, there are 3 intervals...since 14 is not divisible by 3, such an AP does not


exist. so the answer remains 2.

similarly, for atleast 6 terms, v check if 14%4 = 0...since no, the answer is
again 2

for 7 terms 14%5 is not 0, the answer is 2

for atleast 8 terms...14%6 is not 0 so the answer is again 2.

for 9 terms, 14%7 = 0. hence answer becomes 2-1 =1

keep on proceeding like this...the soln wont be so bulky...its been done like this
for ease of understanding...for ease of calculation...see how to proceed...

how many A.Ps are possible such that first term is 1235 and the last term is 3535
and there are atleast...

5. 3 terms

6. 4 terms

7. 5 terms

8. 6 terms

3535 - 1235 = 2300.

2300 = 23x2^2x5^2 . hence, no. of factors = 3x3x2 = 18. (check yesterday's concept
if missed)

how many APs...18.

how many with atleast 3 terms?


since 2300%1 = 0, 18-1 = 17

how many with atleast 4 terms?


2300%2 = 0, hence 17-1 = 16
how many with atleast 5 terms?
2300%3 =/ 0, answer remains 16. where =/ means not equal to

how many with atleast 6 terms?


2300%4 = 0, 16-1 = 15

how many with atleast 7 terms?


2300%5 = 0, 15-1 = 14

how many with atleast 8 terms?


2300%6 =/ 0 , answer remains 14

how many with atleast 9 terms?


2300%7 =/ 0, answer remains 14

and so on...

to check for atleast n terms, v need divisibility till n-2...i'll appreciate if u


dont cram...but understand it...i've neva learnt it...its jus an observation...

so, the answers to above 3 questions wud be 17,16,16,15.

finally...

How many values of a are possible if x^2 + ax + 2400 has...

9. integral roots

10. roots which are natural nos.

this is an actually tricky problem...sad that nobody came up with this...see...

concept:

to solve ax2 + bx + c =0,

we break it as ax2 + mx + nx + c = 0, such that m*n = a*c.

here, a*c = 1*2400 = 2400. so v need to find in how many ways can 2400 be
expressed as product of 2 nos. every such pair of nos. will give a new value of
the coefficient of x.

2400 = 2^5 x 5^2 x 3

no. of factors = 6x3x2 = 36.

but these factors have to exist in pairs...e.g when v use one factor 2 (to express
2400 as 2x1200) the other factor...1200 is automatically used...so total pairs
possible are ...

36/2 = 18.

but relax...this is not it....again...equal no. of negative pairs exist...i.e.


2x1200 corresponds to -2 x -1200. although the product is same as the reqd product
i.e 2400...the sum is different...its 1202 n -1202...n v need to find different
values of sum...hence the answer wud

18x2 = 36 again...

9. integral roots

10. roots which are natural nos.

9. for integral roots...the answer wud be 36


10. values wud be 1/2 the total possible values as negative roots aint
allowed...36/2 = 18 is the answer

Kudos to rockeezee...who got 5 correct answers...so too to junoonmba,vani for


their active participation!!! wud love to see all of u gett'n of the following
correct today...

Today's questions...

smallest no. that has exactly.

1. 20 factors

2. 36 gactors

3. 30 factors

how many APs are possible such that the first term is 2454 , last term is 4254 and
there are atleast...

4. 5 terms

5. 7 terms

6. 10 terms.

how many of the factors of 640000 have

7. perfect square roots.

8. perfect fourth roots

9. perfect cube roots

how many different values can 'A' take if x2 + Ax + 2500 has ...

10. integral roots

11. negative roots.

12. non-negative integral roots.

13. find the no. of factors 15! has (here comes the season of fresh
concepts...which'll derive from older ones...he he)

14. find the no. of factors 18! has


Minimum value that A can take if x2 + Ax + 900 has...

15. Integral roots.

16. Negative integral roots.

17. Positive integral roots.

this is a long post...with several concepts and a wide variety of questions...i


believe it'll take some time to grasp, solve n discuss the concept properly...so i
wont post a new concept tomorrow...i've got messages suggesting i shud be a bit
slow....so lets discuss this very concept till v master it...start solving
guys...at any point of difficulty...i'll be there...

Happy solving!!!!!

with due regards


maxximus

========================================================================

What is the smallest number which on multiplying with 12.5464646464646�. would


make the product an integer????

a) 198 b) 1188 c) 2475 d) 12375

There is a concept behind solving such questions.. Would anyone like to try it
out??? (Mostly for Newbies)

Season 2007-2008:

Results awaited: IIMs


XLRI : Selected
MhCET : 99.83 (146/200

==============================
Hie...sorry guys...was out whole day...so a late reply...

before anything else, i shud give the answers...

smallest no. that has exactly.

1. 20 factors---240

2. 36 factors----1260

3. 30 factors----720
how many APs are possible such that the first term is 2454 , last term is 4254 and
there are atleast...

4. 5 terms----33

5. 7 terms----31

6. 10 terms.----29

how many of the factors of 640000 have

7. perfect square roots.----18

great work gk1, rockeezee...here's a more practical approach...640000=5^4x2^10


express this as 25^2 x 4^5. now since 25 as well as 4 are perfect
squares...combination of their products will also be pefect squares...no. of
combinations here wud be (2+1) x (5+1) = 18

8. perfect fourth roots----6

express it as 16^2 x 625^1 x 4...since 16, 625 have perf fourth roots...so will
their cominations...i.e. 3x2 = 6 of them.

9. perfect cube roots----8

express it as 8^3 x 125^1 x 10...since 8,125 are perfect cubes...their


combinations wud yield perf cubes...their no. are... (3+1) x (1+1) = 8

how many different values can 'A' take if x2 + Ax + 2500 has ...

10. integral roots----16

2500 = 2^2 x 5^4. hence no. of factors are 3x5 = 15.

no. of pairs = {15 / 2 } = 8. {} = least integer function...plz check...rounded it


off to higher value...the value is odd because of 50 x 50 = 2500...giving only one
combiantion seen both wat...while other combinations give 2 different cominations
when the order is changed...for example...250 x 10 and 10 x 250... but 50 + 50
will give a new sum 100...so it shud be counted.
so the answer in above case wud be {15/2}x2 = 8x 2 =16.

for better understanding...lets take one more example with smaller value of a x c.

e.g. x2 + Ax + 4 = 0. A can +/- (2+2 or 4 +1)...so 4 combinations...while 4 =


2^2...hence having 3 factors...to get the answer, we'll perform the following
opeartn...{3/2} x 2(for negative counterparts) = 4.

11. negative roots----infinitely many

great job rockeezee...its not mentioned negative integral roots...so infinitely


many... i know many of u tuk out answer bcoz of faith in my typing errors...n must
have thought that i forgot to type integer...well, in that case..the answer is
16/2 = 8. as 0 cannot be a root of above expression.
12. non-negative integral roots----8

13. find the no. of factors 15! has ---- 12x7x4x3x2x2

14. find the no. of factors 18! has---- 17x9x4x3x2x2x2

...great job rock, junoon...try coming up xplanations as well...18! has 16 2s, 8


3s, 3 5s, 2 7s, 1 11, 1 13, 1 17. added one to the no. of times each prime no.
existed and multiplied them...

Minimum value that A can take if x2 + Ax + 900 has----

15. Integral roots---- (-901) while taking -900 x -1 = 900

16. Negative integral roots---- 60...thru 30 x 30

17. Positive integral roots--- -901 thru -900 x -1

kudos to rockeezee,gk1,junoonmba,iyervani!!! who got many right.....

hope this post clears few concepts....wud suggest u all to kindly go thru this
post n the original post once more to cement the concept...

wud love to hear from u guys to make this thread more effective...as per the
feedbacks so far, am trying to make it slow n steady...

with due regards


maxximus
===========================
Today's concept....finding squares and close multiplications quickly...

In a hurry guys....so few small concepts that'll help u save sum time...n avoid
cramming....which i've always maintained is the best ill-preparation for cat.

#1 finding squares.

step 1. think of a base which is a multiple of 10 or 100 (whichever nearer to the


no. whose square is to be determined.)

suppose 34^2.

a gud base wud be 30.

express 34 as 30 + 4 (i.e. base + 4)...i'll call this base + deviation

step 2.

ldigit(s) before 0 in base x (no. + deviation) | deviation^2 where | is an


imaginary line separating 2 parts of calculation.

i.e. 3x(34 + 4) | 4^2

i.e 114 | 16.

now the important thing....no. of digits on right side of the imaginary line shud
be exactly same as the no. of zeros in our base..dont forget this!!!!
since our base 30 has only one 0 at the end, v can have only one digit on right
side of the imaginary line....the less powerful digit...thats 6.

and thus, one has to be carried to the left side....so 114 will become 115.

so the expression becomes...115 | 6

hence, the answer is 1156.

look at few more examples for practice...

28^2

base 20

2x(28 + 8 ) | 8x8

= 72 | 64

= 784

106^2

106 + 6 | 6x6

base 100

=112 | 36

=11236 . since 100 has 2 zeroes, there shud be 2 digits on right side of the
line....

103^2

base 100

103 + 3 | 3x3

=106 | 9

= 10609. ensure the no. of digits are exactly same as the no. of zeroes in
base....so expressed 9 as 09....a 2 digit no.

312^2

base 300

3x(312 + 12) | 12^2

3x324 | 144

972 | 144

= 97344....1 of 144 carried.


smart ways of using this method....

98^2. now if v take base 100, v can avoid multiplication by 9.

so base 100.

98 - 2 | -2^2

= 96 | 04

= 9604....funny ha?

197^2

base 200

2x(197 - 3) | -3^2

=388 | 09

= 38809.

#2 : finding product of nos when they lie nearby.

all digits before 0 in base x (first no. + deviation of second no.) | deviation of
first no. x deviation of second no.

rest, everything is same...

examples...

27 x 22 =

base 20

2x(27 + 2) | 7x2

= 58 | 14

= 594

103 x 108

base 100

= 103 + 8 | 3x8

= 111 | 24
=11124

97 x 102

base 200

97 + 2 | -3 x 2

=99 | -6

= 9900 - 6

= 9894

197 x 199

base 200

= 2x(197 - 1) | -3 x -1

= 392 | 03

= 39203

37 x 31

base 30

37 + 1) x 3 | 7x1

= 114 | 7

= 1147.

32 x 24

base 30

32 - 8 | 2 x -8

= 24 | -16

= 240 - 16

= 226.

think of few examples, solve them using pen...cross check ur answers...when


accuracy becomes gud, try doing it w/o pen....got lotsa other such methods...lemme
know if its helpful...if so...wud post more...

hope the post helps...practice guys...practice!!!


regards
maxximus===========================

Posts: 320
Join Date: Dec 2006
Location: I:Boston Heart:India
Groans: 51
Groaned at 20 Times in 13 Posts
Thanks: 657
Thanked 1,980 Times in 209 Posts
Send a message via Yahoo to maxximus

Report Bad Post


Re: Concepts...total fundas!! - 31-05-2007, 02:35 AM - Add Post To Favorites
Today's concept : Finding HCF n LCM of typical values.

#1 : to find the HCF, LCM quickly.

i know most of us wud know this....if v have few nos., 20,40,50,80,180...to find
their LCMs, HCF...there's a slightly quick method...

express them in prime nos.

20 = 2^2 x 5
40 = 2^3 x 5
50 = 5^2 x 2
80 = 2^4 x 5
180 =3^2 x 2^2 x 5

now to HCF, see highest power of all prime nos. that are common to all nos.

2 - 2
3- 0
5 - 1

hence hcf is 2^2 x 5 = 20

to find lcm...see highest power of all prime nos across all nos.

2 - 4
3 - 2
5 - 1

hence, lcm = 2^4 x 3^2 x 5 = 1620.

#2 To find HCF and LCM of the form-

2222....30 times.

3333....70 times.

to solve such questions...

for HCF..
take hcf of no. thats being repeated...i.e. hcf of 2 & 3. i.e. 1

take hcf of no. of time these nos. are being repeated...i.e. hcf of 30 n
70...thats 10.

so the hcf is 111...written 10 times.

For LCM...

take lcm of no. thats being repeated...i.e. lcm of 2 & 3. i.e. 6

take lcm of no. of time these nos. are being repeated...i.e. lcm of 30 n
70...thats 210.

so the hcf is 666...written 210 times.

#3...to find hcf and lcm of following form...

2^300 - 1, 8^250 - 1.

the idea is..a^n - b^n is always divisible by a-b. so v need to find highest a-b
that will divide a^n - b^n and smallest term that'll be divisible by a^n - b^n.

express them in a common base.

2^300 - 1 and 2^750 -1.

to find hcf...

take hcf of powers i.e. hcf of 300 and 750...i.e. 150

so the hcf is 2^150 - 1.

to find lcm....

take lcm of powers i.e. lcm of 300 and 750...i.e. 1500

so the hcf is 2^1500 - 1.

Questions :

find hcf and lcm of:

1.2222...250 times and 8888...300 times

2. 333....120 times and 1111...400 times

3. 111...700 times and 9999...200 times.

4. HCF of 33333...200 times. and 777777.....300 times

5. 32^250 -1 & 16 ^ 100 - 1.

6. 81^100 -1 & 243 ^ 200 - 1.


7. 343^150-1 & 2401^100 - 1.

8. 125^200 - 1 & 625^120 - 1.

9. 169^320 - 1 & 32^160 - 1.

for the following questns...

mark 1. - stmt 1 is sufficient.


2- stmt 2 is suff.
3-both are reqd to solve the questn.
4-either is suff.
5-both insufficient.

10. what is the hcf of 5 nos., a,b,c,d,e?

stmt 1 - a=72,b=4,c=6
stmt 2 - d= 8, e = 27.

easy set...hope many wud get all correct...

regards
maxximus
++++++++++===============

Originally Posted by Rockeeze View Post


originally posted by iyervani 30.05.07

One of the smaller sides of a right angled triangle is


(2^2)*(3^3)*(4^4)*(5^5)*(6^6)*(7^7) . It is known that other two sides are
integers.How many triangles of this type are possible.

The approach mentioned was:

the given number is 2^16 * 3^9 * 5^5 *7^7


a^2 = c^2 - b^2 = c-b * c+b
both c-b and c+b be either odd or even ..but here both cannt be odd..
so both even
to write as a product of 2 numbers we can use all powers of 2 excepth the 32th
power..
so no of triangles = (31*19*11*15 -1 )/2

I did not understand the highlighted part...


could someone plz explain this or any other approach to solve the ques.????

well i thought dis ques was worth a discussion in this thread


vanis query was also not ans der..

i too hv the same query


i think the ans should be 17*10*6*8-1/2
number of ways the samller side cn be expressed as a product of two no

hi rock...thanx for bringing this to the thread...

this question has to do with a concept i've already discussed on this


thread...concept of factors...yeah...this question has lotsa twists n turns
attached.

the final answer given by whoever answered it is correct but the xplanation has
more than one mistake...seems the question was solved after seeing the correct
answer...another mal-practice that must be avoided...lets have an elaborate
discussion...

the given number is 2^16 * 3^9 * 5^5 *7^7


a^2 = c^2 - b^2 = c-b * c+b

for convenience, take c-b = t, c + b = u.

both t and u shud be either odd or even ..but here both cannt be odd (since a
pythagorian triplet with hypotenuse = even n rest 2 sides = odd does not exist)
so both even

now a^2 = 2^32 * 3^18 * 5^10 *7^14.

now, since both t & u are even, 2x2 already exists in t * u. so the powers of 2
which can be floated across 2 terms to be multiplies reduces by 2. i.e. 32 -2 =
30.

now, the no. of ways in which pairs can be formed are 31 * 19 * 11 * 15. but lets
not forget out of these cases, there lies a case when t = u....or lets say b = 0.

that particular case shud be discarded...

since the factors wud be used in pairs, answr shud be half the no. of factors...so
the final answer is...

(31*19*11*15 -1 )/2

please feel free to revert..

with due regards

maxximus
==========================

Answers for yesterday's questions...

1.2222...250 times and 8888...300 times----22..50 times, 88..1500times.2.


333....120 times and 1111...400 times---11..40 times, 33..1200 times.

3. 111...700 times and 9999...200 times.----11...100 times, 99..1400 times.

4. HCF of 33333...200 times. and 777777.....300 times --- 11..100 times, lcm is
diff to find... (33333...200 times. * 777777.....300 times) / 11...100 times.

5. 32^250 -1 & 16 ^ 100 - 1----2^50 - 1, 2^10,000 - 1 (nobody got this right)

6. 81^100 -1 & 243 ^ 200 - 1.---- 3^200-1, 3^2000 - 1

7. 343^150-1 & 2401^100 - 1.----7^50 - 1, 7^1800 - 1

8. 125^200 - 1 & 625^120 - 1.---- 5^120-1, 5^2400 - 1


9. 169^320 - 1 & 32^160 - 1.---- 1, prod of 2 nos.

for the following questns...

mark 1. - stmt 1 is sufficient.


2- stmt 2 is suff.
3-both are reqd to solve the questn.
4-either is suff.
5-both insufficient.

10. what is the hcf of 5 nos., a,b,c,d,e?

stmt 1 - a=72,b=4,c=6
stmt 2 - d= 8, e = 27.

answer - option 2. stmt is suffct.


This was the only tricky question in the set...look at statement 2...8,27 are co-
prime nos. so their hcf is 1. hence, hcf of the entire set is 1. so, stmt 2 is
sufficient to answer.

regards
maxximus

Complete Guidance for GMAT and Beyond...Click Here!!


Last edited by maxximus; 01-06-2007 at 01:10 AM.

Digg this Post!Add Post to del.icio.usStumble this Post!


� Quote � Quick Reply Groan! Thank! Multi-Quote: Multi-Quote This Message
The Following 6 Users Say Thank You to maxximus For This Useful Post:
HarshaRocks (02-10-2007), hiran.prashanth (04-02-2009), kaizen_2007 (10-08-2007),
mohit_ranka (02-06-2007), mr.s.k.abhi (31-10-2007), Rockeeze (01-06-2007)
maxximus
View Public Profile
Send a private message to maxximus
Visit maxximus's homepage!
Find all posts by maxximus
Add maxximus to Your Contacts
(#130)
maxximus maxximus is offline
In India until 21st July
Hardcore PaGaL

Posts: 320
Join Date: Dec 2006
Location: I:Boston Heart:India
Groans: 51
Groaned at 20 Times in 13 Posts
Thanks: 657
Thanked 1,980 Times in 209 Posts
Send a message via Yahoo to maxximus

Report Bad Post


Re: Concepts...total fundas!! - 01-06-2007, 01:11 AM - Add Post To Favorites
Quote:
Originally Posted by Rockeeze View Post
hi
was going thru the concept of lcm n hcf
type #2
222.........30 times
333..............70 times

the method is fine, but cud nt find a logic for taking the lcm of 30 n 70
22......... 30 times=2*111..........30 times
n 333......70 times=3*111.........70 times

cud u plz explain the logic??

111...30 times can be written as

111...10 times x 10^20 + 111....10 times x 10 ^10 + 111...10 times.

hence, v can say 111...30 times is divisible by 11...10 times.

similarly..

111...70 times can be written as

111...10 times x 10^60 + 111...10 times x 10^50 +......111...10 times.

hence this is also divi. by 111...10 times.

so, the hcf is 111...10 times.

please note that 10 times is the max no. of times 11..can be written so that it
divides both 30 times n 70 times...

while finding lcm, v need 11... as many times that 11...70 times as well as 11..30
times...so it shud be 111....210 times.

do revert..

regards
maxximus
================================
Today's concept : To solve last digit problems.

Type # 1:

questions of the form...

2003 x 2004 x 4235161006 x 432657178001 x 42315098002 x 423087004

concept:

Last n digits of any product depends on the product of last n digits. so just
multiply last n digits of each term...find the product, take last n digits of the
product n multiply it with the next term...continue this for all terms.

suppose v had to find last digit of the product above...

so multiply last digits.


3 x 4 = 12. dont worry abt 1 in 12. just remember 2 and multiply it with next no.
2 x 6 = 12. so 2, 2 x 1= 2, 2 x 2 = 4, 4 x 4 = 16.
so, the ans is 6.

for last 2 digits:

03 x 04 x 06 x 01 x 02 x 04 = 76

for last 3 digits:

003 x 004 x 006 x 001 x 002 x 004 = 576.

trust me, last 4 digits wont be asked...as then it becomes bulky...questions in


cat are tricky...

Type # 2:

of the form...last digits of 432^43567. i.e base ^ power

i know most of us know this concept...wud take it concisely...

look for the variation in last digit of higher powers of last digit of the
base...i.e. 2 here.

2^1 = >2
2^2 => 4
2^3 => 8
2^4 => 6
2^5 => 2

so we can say that 2,4,8,6 will keep repeating...no. of different digits that the
last digits of higher powers can take is known as cyclicity. every digit has a
cyclicity.

to find last digit, find last digit of :

(last digit of base) ^ (power % cyclicity of last digit)

when, power % cyclicity of last digit = 0,

take,

(last digit of base) ^ (cyclicity of last digit)

in the above example...

432^43567.

2^ (43567 % 4) = 2^3 = 8 answer.

with little practice....u can do all such questions mentally.

Type # 3

Find last 2 digits of 34291^201.


when the last digit is 1,9,0,5...try finding a pattern in last 2 digits...u'll get
one...then solve the question accordingly...

last 2 digits of 91^1 = 91


last 2 digits of 91^2 = 81
last 2 digits of 91^3 = 71
last 2 digits of 91^4 = 61
.
.
.
.
.
.
.
last 2 digits of 91^10 = 01
last 2 digits of 91^11 = 91

see, we again got 91 as last 2 digits...so v can say that the cyclicity of 91 for
last 2 digits is 11 -1 = 10

so the answer shud be 91.

lets take one more example...

find last 2 digits of


(49)^(37)^(38 )^(39)...(3700)

see...

last 2 digits of 49^1 => 49


last 2 digits of 49^2 => 01
last 2 digits of 49^3 => 49
last 2 digits of 49^4 => 01.

can v say that for all odd powers, answer wud be 01?
yes!

since the power of 49 is odd...the answer shud be 01.

what if the question was

(49)^(37)^(38 )^(39)...(3700)%100 = ?
answer wud be 01%100 = 1

(49)^(37)^(38 )^(39)...(3700) % 20 = ?
answer wus be 01 % 20 = 1.
bcoz, to c remainder with 20, v need last 2 digits only.

(49)^(37 )^(38 )^(39)...(3700) % 10


=> 49 % 10 = 9
this method might become tedious when the last 2 digits are unfriendly...like 37,
82 etc. but i have neva seen such figures appearing in cat...to solve such
figures, v need euler's or binomial...ill be taking it when v discuss
remainders...today's questions are based on the above concept only..

Questions for today..

1. last digit of 3677^400 - 689^84

2. last digit of 11^11 + 12^12....1000^1000.

3. Last 2 digits of : 233527 x 54725 x 64535379 x 64536247 x 63546342 x 435272599


x 7454453279 x 63546337 x 543624334547 x 74547459 x 7454373 x 6545347359 x
735473451...do i need to say....its tricky...not lengthy!!

4. no. of zeroes at the end in 1^1x2^2x3^3x...250^250.

5. no. of zeroes at the end in 1! x 2! x 3! x...25!

6. Last non-zero digit of 25!

7. Last non-zero digit of 1! x 2! x 3!...15!

Find last 2 digits of...

8. 81^(371)^(372)^...(400)

9. 11 ^ (25)^(31)^(41)...(1001)

10. 7 ^ 2501.

11. 3^2537837.

Hope the post helps...

with warm regards


maxximus
======================

hie....answers...!!

1. last digit of 3677^400 - 689^84---- 0

2. last digit of 11^11 + 12^12....1000^1000.----nobody got this right

3. Last 2 digits of : 233527 x 54725 x 64535379 x 64536247 x 63546342 x 435272599


x 7454453279 x 63546337 x 543624334547 x 74547459 x 7454373 x 6545347359 x
735473451...do i need to say....its tricky...not lengthy!!----nobody got this
right.

4. no. of zeroes at the end in 1^1x2^2x3^3x...250^250.----try again

5. no. of zeroes at the end in 1! x 2! x 3! x...25!----56


5-9...5 nos. one 0 each = 5
10-14...5 nos. two 0 each=10
15-19...5 nos. three 0 each=15
20-24...5 nos. four 0 each=20
25...1 no. with 6 0s = 6

total = 56

6. Last non-zero digit of 25!----try again

7. Last non-zero digit of 1! x 2! x 3!...15!----try again

Find last 2 digits of...

8. 81^(371)^(372)^...(400)---81

9. 11 ^ (25)^(31)^(41)...(1001)----51

10. 7 ^ 2501.----07

11. 3^2537837.----63

kudos to ramkrishnas, jhajee...who got many correct answers.

questions needed to be discussed further....

1. last digit of 11^11 + 12^12....1000^1000

2. Last 2 digits of : 233527 x 54725 x 64535379 x 64536247 x 63546342 x 435272599


x 7454453279 x 63546337 x 543624334547 x 74547459 x 7454373 x 6545347359 x
735473451

3. no. of zeroes at the end in 1^1x2^2x3^3x...250^250.

4. Last non-zero digit of 25!

5. Last non-zero digit of 1! x 2! x 3!...15!

try these questions...discuss them on the thread...few of them are a bit lengthy
but very useful from practice / concept point of view...i've deliberately made
them long...to let everybody understand wat happens to the last digit when a no.
is multiplied with 15,20,25 etc. take ur time to solve...as i said...this set is v
useful and wud bring in lotsa new concept as u solve it..

with warm regards


maxximus[/quote]
=========================
Answers...

1. last digit of 11^11 + 12^12....1000^1000


Ans 3

from 1^1 till 10^10, we'll get 1,6,3,6,5,6,7,4,9,0. the order will change but the
set will be of these 10 digits only...just an observation!!

so, we're adding the bove sum (1000-10 ) /10 = 99 times...so the last digit shud
be last digit of 47 x 99 = 3.

2. Last 2 digits of : 233527 x 54725 x 64535379 x 64536247 x 63546342 x 435272599


x 7454453279 x 63546337 x 543624334547 x 74547459 x 7454373 x 6545347359 x
735473451
Ans - 50

there's one 25 n one 2...that product will make the last 2 digits 50. now 50 x any
odd no. will always leave 50 as last 2 digits...n all the other nos. are odd

3. no. of zeroes at the end in 1^1x2^2x3^3x...250^250.


Ans 8125

5s... 5 ( 1+ 2+....50) = 6375


25s...25(1+2+..10) = 1375
125s...125(1+2) = 375

total = 8125

4. Last non-zero digit of 25!


Ans - 5

keep multiplying last digits...not a matter of more than 90 secs.

twists:

last non zero digit of 14! = 2..so that of 15! shud be (odd no.)2 x 15 => 8, after
excluding 0... odd no. before 2 bcoz v know the no. shud be divisible by 4...an
even no. before 2 wud prohibit that

last non zero digit of 19! = 6..so that of 20! shud be 2 x 2 => 4, after excluding
0.

last non zero digit of 24! = 6..so that of 25! shud be 25 x (odd no.)6 => 5, after
excluding 0. odd no. for the same reason.

5. Last non-zero digit of 1! x 2! x 3!...15!


ans 6

same approach as above

kudos to sumi,rock,saurabh...who got many correct answers!!!!

questions for today....

find remainder when divided by 100...

1. 1^1 + 11^11 + 111^111 +.... thousand ones^thousand ones

2. 2^20,000

3. 1^1 + 21 ^21 + 31^31 + ....1001^1001

4. 5^5 + 15^15 + 25 ^25 +...125^125

5. 9^9 + 99^99 +.....hundred 9s ^ hundred 9s


happy solving!!!!

regards
maxximus
========================

hi everybody...sorry but no sorry for a deliberate late reply...am overwhelmed by


the active participation out here....so decided to stay away till the heat
settles....am very happy that so many posts discussed today's questions...n v had
a lot of drama before managing all the correct answers...

this has been a special experience...because it's for the first time that all the
questions in the set have been rightly answered...i hope the trend continues...

nothing wud please me more than to copy-paste solution by others using my methods
suggested here...today my job looks easy...n am a happy man

wud request everybody to pour in their self-discovered / short cut methods....

find remainder when divided by 100...

1. 1^1 + 11^11 + 111^111 +.... thousand ones^thousand ones ----90

The remainder when divided by 100, depends upon the last two digits...

So the last two digits of the expression are

01 + 11 + 11 +....... = 01 + 999*11(last two digits of it ) = 01+89 = 90

2. 2^20,000----76

2^20000=(2^4)^5000=16^5000

Now the cyclinity of 16 for the last two digits is 5....

So the answer is 16^5= 76

3. 1^1 + 21 ^21 + 31^31 + ....1001^1001----90 (thanx for noticing missing


11^11...ya i know i have a history of typo...still...as rock said...its always
safe to solve wats printed!!)

last two digits of the expression is given by

(01 + 21+ ..... +91 )+ (01+11 + 21 +...... +91 .... 10 times(till 991) + 01 (for
1001)

449 + 460 *9 + 01 = 449 + 4140 + 01 = 4590

so the answer is 90
4. 5^5 + 15^15 + 25 ^25 +...125^125----25

every expression gives 25 as last two digits


25 + 25 +...... 13 times....= 25*13=25

5. 9^9 + 99^99 +.....hundred 9s ^ hundred 9s----90

89 + 99*99 = 89 + 01 = 90

kudos to MOHIT, ROCK, SAURABH, SUMI, JHA for their correct answers as well as for
the healthy discussions....keep up the spirit guys!!!!

regards
maxximus
=====================
Today's concept : Mastering Data suffieciency.

With no. of options for every question increased to 5, the expected answer set for
D.S. is...

1. Statement 1 alone is sufficient


2. statement 2 alone is sufficient.
3. both statements together can answer the question but none of them is alone
sufficient.
4. either statement is sufficient.
5. both statements are insufficient.

wud like to share my approach towards D.S.

first n the mother of all other ...see if statements 1 is sufficient.

if yes... answer shud be 1 or 4.


............now check if statement 2 is sufficient.
..................if yes, option 4 is correct.
...........................if no, option 1 is correct. dont see if both together
can answer the que.

if no....answer can be 2,3 or 5


...........check if stmt 2 is sufficient.
.................if yes, option 2 is correct. dont see if both together can answer
the que.
..................if no, answer shud be 3 or 5.
.......................check if both 1 & 2 together lead to a unique solution.
.............................if yes, option 3 is correct
....................................if no, option 5 is correct.

do not forget...

1). The answer shud be unique...even 2 cases suggest that the statement is not
suffiecient.

2). The answer shud be consistent...if the answer is no...it shud be always
no...if yes...always yes.

3). Both statement can give different answers...they're independent till used
together...so even if the answers r different...the answer wud optn 4 if each
statement satisfies above 2 conditions independently.

4). at times u'll see that question can be answered using any one option as well
as both options...in such a case, answer wud be 1 or 2 and not 3.

5) the most important rule....everytime u're able to sort a D.S. question


easily...check twice...there must be a catch sumwhere...

Questions for today...

1. Statement 1 alone is sufficient


2. statement 2 alone is sufficient.
3. both statements together can answer the question but none of them is alone
sufficient.
4. either statement is sufficient.
5. both statements are insufficient.

1. is X positive?

stmt 1. x^2 - 5x + 10 = 7
stmt 2. sqrt (x) is real.

2. is X even?
stmt 1. 7x + 5y = even
stmt 2. 3x + 38y = odd.

3. three packages combinedly weight 60 kg. what is the weight of heaviest


package ?

stmt 1. one package weighs 10 kg


stmt 2. one package weighs 32kg

4. is X > Y?

stmt 1. 2/5 X > 3/8 Y


stmt 2. 3/7X > 15/29 Y

5. If p^q = 289, (p-q)^(q/2) = ?

stmt 1. p > q
stmt 2. p is not = 1.

6. for above questn.

stmt 1. p>q
stmt 2. p is prime.

7. are sum goats not cows?

stmt 1. all cows are lions


stmt 2. all lions are goats.
8. is the no. of flowers in the garden = 3576876767576654544334538?

st 1. no. of flowers in a row = no. of flowers in a column.


st. 2 no. of rows = no. of columns.

9. for above question

st 1. no. of flowers in a row = no. of flowers in a column.


st 2. no. of flowers are even.

10. x = ?

st. 1 x^2 - 5x + 6 = 0
st. 2 x^3 - 8 = 0

Happy solving!!!

regards
maxximus

==========================
is X positive?

stmt 1. x^2 - 5x + 10 = 7
stmt 2. sqrt (x) is real.

ans. 1...bcoz both roots for st 1 are positive...so the answer wud be 'yes'
always....while for stmt 2, answer wud be yes for all real vaues of x except 0.
since, 0 is very much real but non-positive...so 2 is insufficient.

2. is X even?
stmt 1. 7x + 5y = even
stmt 2. 3x + 38y = odd.

ans.2...bcoz 38y is even...so 3x must be odd for the sum to be odd. while in stmt
1, x n y are either both even or both odd...so inconsistency..

3. three packages combinedly weight 60 kg. what is the weight of heaviest


package ?

stmt 1. one package weighs 10 kg


stmt 2. one package weighs 32kg

ans.2...if one package is weighing more than half the total...it's gotto be
heaviest.

4. is X > Y?

stmt 1. 2/5 X > 3/8 Y


stmt 2. 3/7X > 15/29 Y
to be discussed further...with xplanations...

5. If p^q = 289, (p-q)^(q/2) = ?

stmt 1. p > q
stmt 2. p is not = 1.
to be further discussed...few of u got the correct answer but with a fallacious
approach...xplanations required!!
6. for above questn.

stmt 1. p>q
stmt 2. p is prime.
to be further discussed...few of u got the correct answer but with a fallacious
approach...xplanations required!!

7. are sum goats not cows?

stmt 1. all cows are lions


stmt 2. all lions are goats.
ans.5...due to one possible case when all lions are cows n all goats are lions..in
this case the answer wud be NO. otherwise its yes...inconsistency...hence option
5.

8. is the no. of flowers in the garden = 3576876767576654544334538?

st 1. no. of flowers in a row = no. of flowers in a column.


st. 2 no. of rows = no. of columns.

9. for above question

st 1. no. of flowers in a row = no. of flowers in a column.


st 2. no. of flowers are even.
nobody got these right....wud like u guys to think more...

10. x = ?

st. 1 x^2 - 5x + 6 = 0
st. 2 x^3 - 8 = 0
ans.3...bcoz st1 will give 2 values....2,3 so alone not suff. st1 will give 3
values...2, other complex...so not sufficient...but using both, 2 is common...so
using both options, v can answer.

kudos to mohit,rock,sumi,dranzer,cat2007,apjonline,junoonmb a for their many


correct answers and healthy discussions....

questions to be further discussed

1. is the no. of flowers in the garden = 3576876767576654544334538?

st 1. no. of flowers in a row = no. of flowers in a column.


st. 2 no. of rows = no. of columns.

2. for above question

st 1. no. of flowers in a row = no. of flowers in a column.


st 2. no. of flowers are even.

3. If p^q = 289, (p-q)^(q/2) = ?

stmt 1. p > q
stmt 2. q is not = 1. (edited...made q is not= 1, hint for all...whether its p not
=1 or q not = 1, answer remains unchanged)

4. for above questn.


stmt 1. p>q
stmt 2. p is prime.

5. is X > Y?

stmt 1. 2/5 X > 3/8 Y


stmt 2. 3/7X > 15/29 Y

i hope we get them all soon...there's sum catch in every question....so come up
with ur answers as well as xplanations...

regards
maxximus
========================

Hie...we've seen a lot happening over the set of 10 D.S questions....later reduced
to 5...there have been right approaches, wrong answers as well as wrong approaches
and right answers (lets hope v carry this luck to cat..!!!) few correct approaches
have been given up...and few wrong approaches have been appreciated....high time i
shud give a solution to the more trickier 5...

i request everybody here to look at the solution very carefully...

1. is the no. of flowers in the garden = 3576876767576654544334538?

st 1. no. of flowers in a row = no. of flowers in a column.


st. 2 no. of rows = no. of columns.

to ensure that the no. of flowers shud be a perfect square, stmt 1 is enough...try
making a grid without equal rown n columns but equal no. of flowers...u'll fail...

so, if the given figure is a perfect square, this stmt becomes insufficient as
there are other perfect squares also. but if its not a perfect square, then this
statement becomes sufficient n v get a consistent answer with this stmt that the
given no. is NOT the no. of flowers in the garden. since, the given no. ends in 8,
its not a perf square and thus, not the no. of total flowers in the garden..so v
get a consistent answer...

come to stmt 2...no. of columns = no. of rows does not necessarily mean that no.
of flowers are also equal in each column n row...think...their can be jus plants
with no flowers etc. try imagining...u'll understand...so this stmt can give yes
as well as no as the answer...so this stmt is insufficient.

so, the answer is 1.

2. for above question

st 1. no. of flowers in a row = no. of flowers in a column.


st 2. no. of flowers are even.

stmt 1 is sufficient...as stated above.

stmt 2 is insufficient as the given no. is even but there are many even nos. and
the given no. of flowers may/may not be the correct value...
so, the answer is 1.

i hope i was justified in re-stating this question many of us cud still not get
the correct answer.

3. If p^q = 289, (p-q)^(q/2) = ?

stmt 1. p > q
stmt 2. q is not = 1. (edited...made q is not= 1, hint for all...whether its p not
=1 or q not = 1, answer remains unchanged)

4. for above questn.

stmt 1. p>q
stmt 2. p is prime.

well....mohit did a great job out of these questions....the answer is 5 for both
the questions...

important:
always see if it's been mentioned whether the values are I,N,W,R or nothing has
been mentioned...had there been an additional information that p,q are N, both the
questions wud have been answerable...

5. is X > Y?

stmt 1. 2/5 X > 3/8 Y


stmt 2. 3/7X > 15/29 Y

well..well...here v come...mother of all problems...see carefully.

stmt 1 reduces to X/Y > 15/16. now if X = 15.5, Y = 16, answer is NO. but if X =
17, Y =16, answer becomes yes...so, this stmt is insufficient...

now the important 1...

stmt 2 reduces to X/Y > 35/29. since the ratio is always > 1, this statement is
sufficient.

what about negative values of X n Y? please understand...the form given to us is


3/7X > 15/29 Y and not X/Y > 35/29...so if v r assuming some -ve values of X n Y,
they shud hold true for the main form as well.

if v assume that X = -36, Y = -29,

X/Y > 35/29: will hold true as the value becomes 36/29 which is > 35/29 but this
gives X < Y. but are v allowed to conclude here that this statement is
insufficient? NO!!...v must check for the main for given...

3/7X > 15/29 Y @ X = -36, Y = -29, becomes...

-108/7 > - 15 does this hold gud??? NO!!! that means X will always be > Y if the
main condition has to hold true!!
so, the answer is 2

important: in inequalities, think twice before taking ratios, cancelling similar


terms, sending denomainator to right side or bring numerator of right side as
denominator in left side....when v do that, signs come into picture...as now 2
negative values will make a positive value...n we'll be comparing 2 positive
values....whereas had v not done the shifting, both values wud have been negative
n we wud have been comparing 2 negative values....n remember, when v change signs,
inequality shud reverse...n when v take ratio...the inequality is not being
reversed.

e.g.

-4 < -3 this is correct!!

but, 4/3 < 1 is not correct!! (specially for u mohit bhai...)

thats y...maths is beautiful...n the numbers...wonderful!!!

wud like to hear from u guys


with due regards
maxximus

ps sorry for so many bolds...but so much in the post was so much important that
cudnt avoid...

=========================
MORE D.S QUESTIONS:

1. is x > y?

st.1 3x = 2k
st.2 k = y^2.

2. is X/12 > y/40?

st1. 10X > 3Y


st2. 12X< 4Y

3. Are two triangles congruent?

st.1 both are right trangles


st 2 both have same perimeter

4. is quadrilateral ABCD a rectangle?

st1. angle A = angle C = 90


st2 AB is parallel to CD

5. mohit drinks glycodin when he's upset. is he upset?

st.1 he's drinking glycodin


st.2 he's not drinking glycodin.

6. if x = k, is x^3 + ax^2 + bx =0?

st.1 a = 0
st.2 -b = k^2

7. a dozen eggs cost 90/- in jan 1980. did a dozen egg cost more than 90/- in jan
1981?

st.1 in jan 1980, the average worker had to work 5 mins to pay for a dozen.
st.2 in jan 1981, the avrg worker had to work 4 mins to pay for a dozen

8. i started for my office, half an hour late than everyday. by how many minutes
was i late for the office today?

st.1 i travelled at 4/5th my usual speed.


st.2 i tuk a root that was 4/5th the usual distance.

9. Rock bought X flowers for Y dollars (X and Y are integers). had he purchased 20
more flowers, he wud have got all for T dollars and saved 10 cents per dozen. how
many dollars did he spend earlier? (remember, questions on flowers have a
history...the history continues...have a tuf time solving this one!!!)

st1. T = 2
st2. he got 10% discount in second case.

10. sumi lived for less than 100 years. in which year was she born?

st. 1 in 20th century, she said he was x years old in the yr x^2.
st. 2 she lied while making statement 1.

happy solving

regards
maxximus
=========================

hey...seek solutions so that v move to the next concept soon...has everybody


noticed these questions?

MORE D.S QUESTIONS:

1. is x > y?

st.1 3x = 2k
st.2 k = y^2.

2. is X/12 > y/40?

st1. 10X > 3Y


st2. 12X< 4Y

3. Are two triangles congruent?

st.1 both are right trangles


st 2 both have same perimeter

4. is quadrilateral ABCD a rectangle?

st1. angle A = angle C = 90


st2 AB is parallel to CD

5. mohit drinks glycodin when he's upset. is he upset?

st.1 he's drinking glycodin


st.2 he's not drinking glycodin.
6. if x = k, is x^3 + ax^2 + bx =0?

st.1 a = 0
st.2 -b = k^2

7. a dozen eggs cost 90/- in jan 1980. did a dozen egg cost more than 90/- in jan
1981?

st.1 in jan 1980, the average worker had to work 5 mins to pay for a dozen.
st.2 in jan 1981, the avrg worker had to work 4 mins to pay for a dozen

8. i started for my office, half an hour late than everyday. by how many minutes
was i late for the office today?

st.1 i travelled at 4/5th my usual speed.


st.2 i tuk a root that was 4/5th the usual distance.

9. Rock bought X flowers for Y dollars (X and Y are integers). had he purchased 20
more flowers, he wud have got all for T dollars and saved 10 cents per dozen. how
many dollars did he spend earlier? (remember, questions on flowers have a
history...the history continues...have a tuf time solving this one!!!)

st1. T = 2
st2. he got 10% discount in second case.

10. sumi lived for less than 100 years. in which year was she born?

st. 1 in 20th century, she said he was x years old in the yr x^2.
st. 2 she lied while making statement 1.

and sunil natraj has given an additional question...sum of 100 terms of n(n+2)

do start com'n up with solutions...

@sunilnatraj...ur expertise wud be helpful bro...

@focused2007...u're welcome to the thread!!

happy solving

regards
maxximus
+========================
Hie...here's the solution to the last set of D.S questions...sorry for the delay!!

1. is x > y?
st.1 3x = 2k
st.2 k = y^2.
Ans - 5...easily, neither is alone sufficient and using both, v get x = 2y^2/3
which may or may not be greater than y. e.g. y=1 => x = 2/3 hence answer wud be
NO. and y = 3=> x=6, now the answer wud be YES. so v cant answer this ques.

2. is X/12 > y/40?


st1. 10X > 3Y
st2. 12X< 4Y
Ans � 1...the question trims down to if x/y > 3/10. using st 1, we consistent
YES,using st2 v dont get consistent yes or NO as st2 is X/Y<1/3..which may or may
not be >3/10. so, statement 1 is alone sufficient, st2 is not. Dont worry abt
negative values....as xplained in the last set!

3. Are two triangles congruent?


st.1 both are right trangles
st 2 both have same perimeter
Ans - 5...two such sets do exist...e.g. 3,4,5 and 2 , 4.8 , 5.2 i.e same
perimeter,rt triangles but incongruent.

4. is quadrilateral ABCD a rectangle?


st1. angle A = angle C = 90
st2 AB is parallel to CD
Ans - 1..mohit's solution

Drawing two diagonally opposite 90 degrees lines on paper, will give us a figure
like the following
................C
..............7 (seven means a right angled joint, it should be taken as inverted
L)

...L
A
Now, extending these two points A and C to create the the quardilateral ABCD,
gives us a rectangle.

statement 2 is NOT sufficient alone,as AB || CD, but there may not be rectangle,
if BC is not || AD. So the answer is 1

5. mohit drinks glycodin when he's upset. is he upset?


st.1 he's drinking glycodin
st.2 he's not drinking glycodin.
Ans - 2...welll...mohit knows exactly when he drinks glycodin...so he's the best
person to answer this...his solution:
statement 1 is NOT sufficient alone as mohit MAY drink glycodin when he is NOT
upset too.
statement 2 is sufficient as he is not drinking glycodin means he is NOT upset
:smile:. So the answer is 2

6. if x = k, is x^3 + ax^2 + bx =0?


st.1 a = 0
st.2 -b = k^2
Ans � 3 Mohit's soln..statement 1 is NOT sufficient alone, as it reduce the term
to k^3 + bx, which might be and might not be equal to zero.
statement 2 is also not sufficient as it reduces the term to ak^2, which again
might be and might not be equal to zero.
considering both statements together, we get 0 in L.H.S, hence the answer is 3

7. a dozen eggs cost 90/- in jan 1980. did a dozen egg cost more than 90/- in jan
1981?
st.1 in jan 1980, the average worker had to work 5 mins to pay for a dozen.
st.2 in jan 1981, the avrg worker had to work 4 mins to pay for a dozen
Ans - 5...v have no idea wat happened to wages of workers. @mohit...bhai...neva
asume...what if i assume that the wages definetly changed? the idea is..wages may
or may not have changed...hence, 5.

8. i started for my office, half an hour late than everyday. by how many minutes
was i late for the office today?
st.1 i travelled at 4/5th my usual speed.
st.2 i tuk a root that was 4/5th the usual distance.
Ans � 5...this is a tricky one...nobody got this right...its more abt language of
the question...see...everywhere the term is 'in reference to usual stuff'...30
mins late from usual time...4/5th usual speed...4/5th usual distance...but look at
the question...by how many minutes was i late for OFFICE...the reference has been
changed from usual timings to office...we donno how punctual i am at my
office...may be i reach 30 mins early everyday...or may be i reach 1 hr late
everyday...hope its clear!!!

9. Rock bought X flowers for Y dollars (X and Y are integers). had he purchased 20
more flowers, he wud have got all for T dollars and saved 10 cents per dozen. how
many dollars did he spend earlier?
st1. T = 2
st2. he got 10% discount in second case.
Ans � 1....nobody got this either....a very simple problem though...rock spent 2
dollars while buying 20 more flowers...so he must have spent <2 dollars
earlier....right? st1 says he spent 2 dollars later...so earlier he must have
spent 1 dollar only...as its given that he spent integral no. of dollars...no
calculation reqd...st1 alone suffct, st2 is not..hence 1.

10. sumi lived for less than 100 years. in which year was she born?
st. 1 in 20th century, she said he was x years old in the yr x^2.
st. 2 she lied while making statement 1.
Ans-1. Mohit gave a beautiful soln...here's wat he wrote..
the only perfect square is 1936 (of 44) in 20th century. hence sumi was born in
1936-44 = 1892.
statement 2, just says that the year is not 1936, and hence its NOT sufficient
alone to tell the age of sumi. so the answer is 1

kudos to mohit,rock,sumi,ankur,apjonline,jha,junoonmba who got many correct


answers!!

up next..remainders

regards
maxximus
=======================================================

Remainder questions can be broadly divided into 2 types...

1). LCM based questions. e.g. a no. leaves remainders 3,2 when successively
divided by 5,6...what will be the remainder when this no. is divided by 30?

2). Power based questions. e.g. remainder when (31)^[(373)^(432)] is divided by 7.

we have already discussed type 1 in two parts...those who have missed it or wish
to revisit the concept may use the link below.

first half

http://www.pagalguy.com/forum/quanti...-fundas-2.html (Concepts...total fundas!!)

second half
http://www.pagalguy.com/forum/quanti...-fundas-5.html (Concepts...total fundas!!)

so what we're left to discuss is type 2 mentioned above...i think there are 3 ways
of solving these questions...v gotto use our own sensibility to c which approach
suits where...

a) Using binomial theorem


b) using cyclicity with remainders
c) using euler's theorem.

for better understanding, i'll divide this post into 3 parts...n will discuss one
approach in each part...

Finding remainders using binomial theorem.

(x+y)^n can be expressed as :

nC0 x^n + nC1 x^(n-1)y^1 + nC2 x^(n-2)y^2 + .......... nCr x^(n-r)y^r + ..... nCn
y^n

concept:
first term, i.e nC0 x^n is the only term that is independent of y and last term,
i.e nCn y^n is the only term that is independent of x. rest, all the terms are
divisible by both x & y.

we'll leverage this property to solve complex problems.

e.g. 28^37 % 9 = ?

see...v can express it as (27 + 1 )^37 % 9. now, since 27 is a multiple of 9, the


only term that'll be independent of 9 will be the last term i.e. nCn 27^0 * 1^n =
1.

hence, the remainder is 1%9 = 1.

important observation:
we know 1^ (any damn thing...even infinity) = 1 and (-1)^(anything) = 1 or -1 with
even and odd values of power respectively. so we'll try getting a form of (nD +/-
1)^N. so that v r ultimately left with (+/-1)^N.

342^423 % 7 = ?

step1:

342%7 = 6.
so the question becomes...

6^423 % 7.

express 6 as 7 - 1.

so the expression becomes...

(7-1)^423.

the only terms independent of 7 is ... -1^423 = -1.


hence, the remainder is -1 + 7 = 6.

523^325 % 7 =?

=> 5^325 % 7.

now, i seek a remainder of 1 or 7-1=6 with a power of 5.

5^1 % 7 = 5....wont work


5^2 % 7 = 4....wont work.
5^3 % 7 = 6...will work.

so, (5^3)^ (325/3) = (5^3)^ (108 ) x 5

= (-1)^108 x 5

= 1x5 =5...the remainder.

529^700000 % 7

= 4^700000

4^3%7 = 1. hence, we'll express the given expression in terms of 4^3

4^3 ^ (700000/3)...c how to save time...700000%3 = (7 + 0 + 0 + 0 + 0 + 0) % 3 =


1. hence (700000-1)/3 will be an integer...dont evaluate its value... bcoz 1^
anything = 1.

hence v have 1^I x 4^1 = 1 x 4 = 4

therefore, the remainder is 4.

(31)^[(373)^(432)] % 7 = ?

31 % 7 = 3

hence,

=(3)^[(373)^(432)]

3^3 = 27 = 28 -1. hence, expression shud be in terms of 3^3

= 3^3 ^ [(373^432)/3]

now treat [(373^432)/3] as a new, different question...

(373^432)%3 = 1.

hence, the term becomes 3^3^(I) x 3^1

I = (373^432 - 1)/3 and is of the form (odd-odd) / odd = even/odd = even for
sure!!

hence, the expression becomes....

(3^3)^(even I) x 3 % 7

= (-1)^even integer x 3 = 1 x 3 = 3

hence, remainder = 3.

case when divisor is a large no.

in such questions, v try to reduce power by increasing the value of base and
bringing it close to a multiple of divisor.

e.g. 2^35 % 61 = ?

v know 2^6 = 64

hence, (2^6)^5 x 2^5

2^6 % 61 = 3.

hence, 3^5 x 2^5

= 3^4 x 3 x 2^5

=3^4 % 61 = 20.

3x 2^5 = 96, 96%61 = 35.

hence, 20 x 35 % 61 = 700%61

= 29, the reqd answer.

hope the concept helps...will shortly post the remaining 2 parts as well...

regards
maxximus

Complete Guidance for GMAT and Beyond...Click Here!!


Last edited by maxximus; 09-06-2007 at 12:46 PM.

Digg this Post!Add Post to del.icio.usStumble this Post!


� Quote � Quick Reply Groan! Thank! Multi-Quote: Multi-Quote This Message
The Following 26 Users Say Thank You to maxximus For This Useful Post:
anshul14 (26-06-2007), atshubha (31-07-2007), daniket (26-06-2009), gautamgomzi
(08-10-2007), gmat_delhi2007 (07-09-2007), harneet9 (21-06-2008), HarshaRocks (02-
10-2007), iyervani (09-06-2007), ketce (22-06-2007), kondapalli (06-09-2007),
kooldud (16-09-2007), mba.yodha (10-09-2008), mohit_ranka (10-06-2007), mukg (14-
06-2009), neha_tyro (25-10-2008), Rockeeze (09-06-2007), Rohan Koshy (31-07-2009),
saurabhgulati (10-06-2007), scorpion_girl (20-06-2007), sgx100 (13-06-2007),
SUMI_08 (10-06-2007), TargettCAT (07-09-2008), varshita (09-06-2007), vivek.anand
(13-06-2007), vyomb (09-06-2007), willrock (30-06-2008)
maxximus
View Public Profile
Send a private message to maxximus
Visit maxximus's homepage!
Find all posts by maxximus
Add maxximus to Your Contacts
(#323)
=============================
Finding remainders using cyclicicty with remainders:

This approach is useful when the divisor is small or at times when it is a factor
of 100.

3^327%7 = ?

3^1 % 7 = 3
3^2 % 7 = 2
3^3 % 7 = 6
3^4 % 7 = 4
3^5 % 7 = 4x3 % 7 = 5
3^6 % 7 = 5 x 3 % 7 = 1
3^7 % 7 = 1 x 3 % 7 = 3

remainder with first power is same as remainder with 7th power...hence v can say
that cyclicity in remainders is 7-1 = 6.

so, 327 % 6 = 3,

hence, effectively, the remainder is 3^3 % 7 = 6

326^524 % 9 =?

326 % 9 = 2

hence, 2^524

now,

2^1 % 9 = 2
2^2 % 9 = 4
2^3 % 9 = 8
2^4 % 9 = 7
2^5 % 9 = 5
2^6 % 9 = 1.
2^7 % 9 = 2

hence cyclicity in remainder = 7-1 = 6.

524 % 6 = 2

hence, the remainder is 2^2 % 9 = 4.

81^502 % 100

81^1 % 100 = 81
81^2 % 100 = 61
81^3 % 100 = 41
81^4 % 100 = 21
81^5 % 100 = 01
81^6 % 100 = 81

hence, cyclicity = 6-1 = 5.

502 % 100 = 2.

so the reqd remainder is same as that with 81^2 = 61.

similarly...this method can be effectively used when remainders are other factors
of 100.
viz, when the factors are 20,25,50,100...knowing last 2 digits wud suffice knowing
the remainders...we've already discussed this concept while discussing
cyclicity...

important: dont try this method when the divisors are complex...viz 37,73
etc...the cylicty wud come very late n calculations will grow cumbersome...when
divisors are complex, there must be sum other catch in the question...look for
that catch...e.g. last type discussed in the binomial method...

regards
maxximus
=============================
Finding remainders using Euler's theorem.
(special thanks to junoonmba for this)

This method is very useful when the divisor and dividend are relatively prime
numbers...

step 1: To calculate euler's no. of a divisor.

euler's no. can be practically taken as cyclicity in remainders by a divisor..

to find euler's no, express the divisor in terms of prime factors...

100 = 2^2 x 5^2.

powers of the prime nos. have no significance...its jus the prime no. that
matters...

euler's no (e for convenience) = divisor x (1-1/first prime factor) x (1-1/second


prime factor) x ... (1-1/last prime factor)

so, for 100, e = 100 x (1-1/2) x (1-1/5) = 100 x 1/2 x 4/5

= 40.

that means e for 100 = 40. or, in other words, 100 divisor will definetly show a
cylicity of 40 in the remainders.

whenever the power of a relatively prime no. will be a multiple of 40, the
expression wud show a remainder 1 with 100.

e.g. 3^120 % 100 = ?


we know e for 100 = 40.
3 n 100 are relatively prime nos.
hence, 3^40 % 100 = 1.

hence 3^120 % 100 = (3^40)^3 % 100 = 1^3 = 1.

7^100 % 45 = ?

45 = 3x3x5

e for 45 = 45 x (1-1/3) x (1-1/5) = 24

hence, 7^24 % 45 = 1

hence, 7^100 % 45 = 1^4 x 7^4 % 45

= 2401 % 45

= 16, the required answer...

with this, we have finished the conceptual part of remainders...now what we seek
is practice...we need to solve a wide variety of problems to improve our reflexes
while choosing the best method for solving a question....

a sincere request to everybody around...please drop in the most difficult


questions on remainders u've eva encountered...let's solve them thru various
approaches and discuss our solutions...for the coming few days...lets know
remainders inside out...

regards
maxximus

+++=======================

Hi puys,
Its been many days since I last posted on PG, but I was occupied with unavoidable
commitments , and now it seems, I have lot of catching up to do...

Firstly solution to QQAD # 49 (Sorry for the delayed post)

As its mentioned in the question that the age of the twins is a teen non-prime no.
hence it can be either 14, 15, 16 or 18....

Probable Ages Sum Product Ages Raveena Thought

14, 14, X 28 + X 196X 7,7, 4X (not satisfy add. criteria)

16, 16, X 32 + X 256X 2,2, 64X (___ DO____)


18, 18, X 36 + X 324X 2,2, 81X (___ DO____)

...............................................3,3 , 36X (___ DO____)

15, 15, X 30 + X 225X 3,3, 25X (satisfies all criteria for X=1)

Thats why the ages are 15, 15, 1....

I think i am clear enough :smile:

Regards,
Mohit

PS - There have been some doubts regarding this question. Kindly read the
conversation between me and vineetvijay, from here onwards..... it will help,
grasping the problem..

http://www.pagalguy.com/forum/quanti...s-day-165.html (CAT 2007: Quantitative


Questions a Day 1 to 50 - The discussions)

P P S : @ Maxximus bro, Please invite problems from other puys, only regarding to
the current-concept-in-consideration..... If there will be many posts comprising
different concepts, we will not be able to learn the concept thorouly, which will
defy the Aim of this thread.

"Democracy, is an anarchy, having multiple dictators"


--Unknown

Moi Blog: http://mohitranka.wordpress.com

Moi Trainee PaGal now...

Digg this Post!Add Post to del.icio.usStumble this Post!


� Quote � Quick Reply Groan! Thank! Multi-Quote: Multi-Quote This Message
The Following 2 Users Say Thank You to mohit_ranka For This Useful Post:
Rockeeze (10-06-2007), rosh! (14-06-2007)
mohit_ranka
View Public Profile
Send a private message to mohit_ranka
Visit mohit_ranka's homepage!
Find all posts by mohit_ranka
Add mohit_ranka to Your Contacts
(#345)
mohit_ranka mohit_ranka is offline
has no status.
Trainee PaGaL

mohit_ranka's Avatar

Posts: 92
Join Date: Aug 2006
Location: Bhilwara
Age: 23
Groans: 12
Groaned at 5 Times in 4 Posts
Thanks: 121
Thanked 59 Times in 40 Posts
Send a message via Yahoo to mohit_ranka

Report Bad Post


Re: Concepts...total fundas!! - 10-06-2007, 06:40 AM - Add Post To Favorites
Quote:
Originally Posted by junoonmba View Post
Solve this...really nice

There are 1000 soliders with a no. on their T Shirts (1,2,3.....999,1000) standing
on a circular track.
Man with no.1 is carrying a sword in his hand. He kills man at no. 2 and pass on
sword to no. 3.
This goes on until we have only one solider on the track.
What will be the no. on his T-Shirt?
If n is the last number on the T-shirt (or n is the total number of people), then
we observe the following pattern

n ___________________ last surviour T-shirt number


1 ___________________ 1
2/3 _________________ 1/3
4/5/6/7 ______________1/3/5/7
2^n/..... /2^(n+1)-1 ____1/3......./2^(n+1)-1

so for n=1023, the surviour will be 1023. and from back tracking from 1022 to
1000, we get for n=1000, the surviour will be 1023-2*23 = 1023-46 = 977

Hope thats correct!!

Regards,
Mohit

Tip: For these kind of questions, where you can find the answer by mechanical
calculations, but n is very large, try to find a relation in the answer and n, or
in other words try to generalize the problem for any value of n and substitute the
original large n in the generalized relation to get the answer..... key thing is
to realize that in which question, generalization will work...

"Democracy, is an anarchy, having multiple dictators"


--Unknown

Moi Blog: http://mohitranka.wordpress.com

Moi Trainee PaGal now...


Last edited by mohit_ranka; 10-06-2007 at 10:34 AM. Reason: Added Tip

Digg this Post!Add Post to del.icio.usStumble this Post!


� Quote � Quick Reply Groan! Thank! Multi-Quote: Multi-Quote This Message
The Following 7 Users Say Thank You to mohit_ranka For This Useful Post:
junoonmba (10-06-2007), martinet (23-06-2008), maxximus (10-06-2007), neha_tyro
(25-10-2008), priyom (27-07-2007), Rockeeze (10-06-2007), vivekr (10-06-2007)
mohit_ranka
View Public Profile
Send a private message to mohit_ranka
Visit mohit_ranka's homepage!
Find all posts by mohit_ranka
Add mohit_ranka to Your Contacts
(#346)
junoonmba junoonmba is offline
out on the town and pigging out!!!!!
Addicted PaGaL
SJMSOM Mumbai

Posts: 1,060
Join Date: May 2006
Location: delhi
Age: 26
Groans: 56
Groaned at 56 Times in 31 Posts
Thanks: 719
Thanked 782 Times in 351 Posts

Report Bad Post


Re: Concepts...total fundas!! - 10-06-2007, 10:36 AM - Add Post To Favorites
@Mohit_Ranka
Yes, you are correct and nice explanation ...........................
Some food for thought...
1.Remainder of 26^16 divided by 125
2. Remainder of 3^94 divided by 125
3.Reamainder of 2^13 divided by 25
4. Remainder of 10^14 divided by 37
5 2^1990%1990

Some phew.. questions for practise.....


============================================
Ok...euler's cannot be applied here since 7 and 28 are NOT relatively prme...
so...
7%28 == 7
7^2 %28 == 21
7^3%28 == 7
7^4 %28 == 21
and so on...this pattern repeats...
so 7 to the power of an even number gives the remainder as 21..

so my ans is 21

=======================

Quote:
Originally Posted by vyomb View Post
puys help me in solving this
stsmt a and b are to be analyzed and answered

1 if qns can be answered by using one of the stmt alone but nt by using the other
stmt
2if the qns cna be answered by using either of the stsmt alone
3if the qns can be answered by using both the stmts together
4if the qns cannot be ansered on the basis of any of the stmts

Zakib spends 30% of his income on his children's education,20% on recreation and
10% on health care.The corresponding percentages for Supriyo are 40%,25% and
13%.Who spends more on children's education?

A.zakib spends more on recreation than supriyo


B.supriyo spends more on helthcare than zakib

hi vyomb...

the answer wud be 1.

i.e. using statement B alone...see the solution...

for expenditure on child..., .3Z > .4S

or, Z/S > 1.3333 for the expense of A to be more...

now use statements...

A says that .2Z > .25S => Z/S > 1.25 which may or may not be > 1.3333...hence st A
is insufficient...

st B says .13S >.1Z ....hence, Z/S < 1.3 therefore, this is definetly less than
1.3333.. so stB will consistently give one answer...NO.

hence, answer is 1, using statement B only...

feel free to revert

regards
maxximus
============================
hie...here's a less than 3 mins solution to 2^1990 % 1990

please see carefully...u can pick a new concept from thsi post...

take divisor as 10 x 199

last digit of 2^1990 = 4.

hence remainder when 2^1990 is divided by 10 is 4.

now lets take 2^1990 % 199

e for 199 = 198

that mean 2^1980 % 199 = 1.

hence, 1 x 2^10 % 199 is effective remainder with 199.

i.e 1024%199 = 29, easily

so with 199, remainder is 29


with 10, remainder is 4

so, v r lookin for a no. that leaves 29 rem with 199 and 4 with 10.

guys who have been following this thread wud know how to solve this part quickly

no. wud be (29 + 199k) such that (29 + 199k) % 10 = 4


(29 + 199k) %10 can be reduced to...

9 + 9k = 4,14,24...

smallest integral value is k=5.

hence the no. v r looking for is 29 + 199 x 5 = 1024

1024 % 1990 = 1024

hence, the answer is 1024.

now that v have solved all the questions with short, practical approaches...can v
practice few more?

solve these...

2^2!^3!^4!...100! %

1. 7

2. 9

3. 5^32 % 5000

4. 2^60 % 130

5. 7^50 % 1001

regards
maxximus

=============================
Maxximus bhai.........Thoda gyaan humari taaraf se :-)

From Number system:-


Concept 1:-
(N^p-1)-1%p gives remainder----0 where p is prime number nd n,p are relativily
coprime.................

Concept 2:-
(p-1)!+1 %p gives remainder-------0 where p is prime number

Questions for practise---------

1 99^6-1%7

2 456^18%19
3 (741^198 -192)%199

4.100!%101

5 (28!+233)%899
========================

hey sumi!

here goes the concept of inverse euler number::

say you have to calculate the remainder of the question::

4^15/17

you can very easily go the traditional way and calculate but the concept of
inverse euler will prove handy and will save whole lot of time.

4^16 / 17 will be 1 (why????)

4^ 15= 4^16*4^-1

thus the question will become 4^16*4^-1/ 17

which is same as 4^-1/17(why?????)

FUNDA::

say if you have N^-b/p then you look for a number "A" such that N*A/p will be 1.
After that the above equation can be written as A^b/p.

See in the above case::


what is the smallest number which should be multiplied with 4 so that remainder
will be 1.
hmmmmmm..pondering over it and you will see 13 is the number.

thus 4*13/17 is 1.
hence equation will reduce to 13^1/17 which is nothing but 13.

cross check by going it the novice way.

some more for practice::

8^828/167

31^78/123

5^26/29

regards!!
marijuana_user.

Want deep insight on GMAT!!

Visit::
www.expertsgloba

===========================
well the formula gives e.n of 100 as 40,but what i have learnt and what i have
seen is that it is better to break up the bigger number in terms of its prime and
then calculate the euler number.I have explained the same case for 77,which by
formula gives e.n as 60 but if u break it up in terms of 6 and 11 then you get the
answer as 30.Cross check it by calculating the remainder for 3^32/77 or say
12^36/77

hi marijuana...its great to c u active on the thread...

abt the discrepancy in the euler's....wud like to throw some light...while


pondering over euler's...even i tried coming up with short-cuts...e.g. i know with
3, 100 wud show cyclicity in remainder after 20 only n not 40..still i believe
both methods are fine...n v can stick to basic euler's to avoid confusion...

1). Technically speaking....even if v come up with some re-arrangement...v shud


not (a euphemism for v font have the authority to) say that euler's no. has
changed...that wud cause confusion as technically euler's no. is wat v get from
x(1-1/p1)(1-1/p2)... where p1 and p2 are prime nos. so when v take 25 n 4...v aint
following eulers as these aint prime nos. hence, it's be better to say that
euler's no. can be taken as 20 for ease of calculation.

2). now when u say bigger nos, there can be many set of bigger nos, for ex. 100
can be broken as 50,2 20,5 etc....each wud give a different e...though i'd take it
for granted that when u said take bigger nos, they shud be mutually prime...infact
highest powers of each prime no. taken independetly..

3). its safe to use euler's relation...remember, when v take e as 40, instead of
20 for 100....v r only taking a multiple of 20...v are not taking a factor of
20...so even if 20 gives correct answer, there's no doubt 40 will always give
correct answer.

great work marijuana....nothing wrong with ur method...its only going to ease the
calculations...n the answers shud always be same using conventional eulers or the
method suggested by u...

regards
maxximus
===========================
Hey max! well nice to c u 2 man! a fauji in d past! great ! m a fighter pilot in
the past ! merit out ! luck had its way. anyways i just wanted to share this::

i have been following PG for a long time now and all this time that i followed it
i was in college.So this is an advice for college goers::
sometimes one tends to get carried away while being online because of so many
things happening on PG(it is this feature of PG which makes it out of the world
and so special) and this tends to waste whole lot of time.So i request you all to
keep a schedule.Dont give preference to Pg over your self study.Once you are
through your self study,browse PG and share the garguntuan amount of
knowledge.Follow certain threads and dont try to glance here,there and
everywhere.like FOLLOWING THIS THREAD SHOULD BE PART OF YOUR SCHEDULE TILL THE VRY
END(excluding the time for self study).Also some times valuable info like
directors interview and all is available on PG so that becomes a must read.

For working people PG is the panacea to all probs(bowing down).As it comes to the
rescue,coz one can browse through it endlessly.

Hope it will help!


I promise to stick to this thread till the very end.

well what i observed::


1. Most of the people active on this thread are from
DELHI(sumi,Junnon,Myself,Max).
2. So why dont plan a meet on this sunday???
===========================

You might also like